LSAT and Law School Admissions Forum

Get expert LSAT preparation and law school admissions advice from PowerScore Test Preparation.

User avatar
 Dave Killoran
PowerScore Staff
  • PowerScore Staff
  • Posts: 5852
  • Joined: Mar 25, 2011
|
#40890
Complete Question Explanation
(The complete setup for this game can be found here: lsat/viewtopic.php?t=15551)

The correct answer choice is (D)

This is a Justify question, and consequently you must find the answer that, when added to the existing rules, forces a single solution for the seven photographs. We previously discussed the importance of I and L, and two of the answers—(C) and (D)—include I or L, providing a good point of attack for this question.

One of those two answer choices, (D), turns out to be correct. When L is first, then I must be second, and M must be third. Because L is first, F must be seventh. The H :longline: GK sequence must then fill in the fourth, fifth, and sixth positions, leading to the following solution:

pt59_d09_g2_q8.png

Note that answer (C) also places I second, but does not fix the position of L or M.
You do not have the required permissions to view the files attached to this post.
 SherryZ
  • Posts: 124
  • Joined: Oct 06, 2013
|
#13060
Hi there, thank you very much for your help! It truly means a great deal to me!

Dec 2009 LSAT, Sec 1 LG, Game 2, Q8:

I got this question correctly but it is time-consuming. Could you tell me how to solve this question quickly?

Also, I find out that I am really bad at doing this kind of global questions like "Where is FULLY DETERMINED if which one is true...". Could you tell me how to solve this type of "Fully Determined" questions quickly??

I sincerely appreciate your help!

---Sherry
User avatar
 KelseyWoods
PowerScore Staff
  • PowerScore Staff
  • Posts: 1079
  • Joined: Jun 26, 2013
|
#13074
Hi Sherry!

For questions like this, I focus on the answer choices that involve variables whose placement forces where other variables go as well as positions that once taken, force variables into certain places. I also think about rules like the IL/LI rule. There are probably multiple placements which would narrow down I and L to two positions. But to determine where every variable goes, I need a condition that tells me whether it's IL or LI.

An answer choice like (D) stands out to me because it involves that 1st position. Once I put L 1st, I must be second so I don't have to worry about the IL/LI rule anymore. Also, filling the 1st position, forces F 7th, so the ends of my linear set up are filled. With I and L first and second, there's only one slot left for M to go in: 3rd. Then I have three slots left and three variables that are part of a chain (H>GK) so I know where everyone needs to go.

None of the other answer choices appear to have that much affect on my other variables. And they especially don't tell me whether it's IL or LI. For instance, answer choice (C) says I is 2nd. But if I is 2nd, L could be 1st or 3rd, so that doesn't help me.

Hope this helps!

Best,
Kelsey

Get the most out of your LSAT Prep Plus subscription.

Analyze and track your performance with our Testing and Analytics Package.